OMSITE 2006

Réussis tes devoirs et examens dès maintenant avec Quizwiz!

36. Recurrent sialocoele formation after emergency room repair of a deep facial laceration is best handled by: A. multiple aspirations, pressure dressings, and antisialagogues. B. superficial parotidectomy. C. reexploration of the wound, cannulation of Stensen's duct, and repair of the parotid capsule. D. low dose radiation.

Answer: A & B Rationale: Sialoceoles that repeatedly reform, even after aspiration, and pressure dressings, and antisialogogues may indicate a parotid capsular or Stensen's duct injury that requires early reexploration and repair. Sialocoele formation within 48 hours can indicate an injury to the salivary ductal system and exploration of the injury is needed. The patient also needs to have antibiotic coverage with penicillin or a cephalosporin. Reference: Fonseca RJ and Walker RV, "Management of soft tissue injuries", Oral and Maxillofacial Trauma, vol 1, pg. 642. WB Saunders, 1991

68. Fracture of alveolar process during the extraction of a tooth is usually due to: A. use of excessive amounts of uncontrolled force. B. presence of adjacent crowded teeth. C. periapical infection. D. expansion of surrounding alveolar bone.

Answer: A RATIONALE: If the alveolus is fractured, dissect the alveolus off the root, replace the periosteum and alveolus which will act as a vascularized graft. If soft tissue is inadvertently dissected from the alveolus, the segment of bone will likely undergo necrosis. If the maxillary tuberosity is fractured, and the tooth cannot be dissected from the alveolus, replace and splint for 6 - 8 weeks. Later surgically extract the tooth. If the tuberosity is completely avulsed without soft tissue pedicle, then smooth the remaining bony edges and close, checking for oral antral communication. Reference: Peterson, LJ; Prevention and Management of Complications in Peterson, Ellis, Hupp, Tucker. Contemporary Oral & Maxillofacial Surgery 4th edition. pp 228-230., Mosby, 2003 USA. Peterson LJ:Contemporary Oral and Maxillofacial Surgery, 3rd edition Ed. Pg. 261 - 262. Mosby, Missouri

68. Which of the following has a favorable aesthetic impact on an implant-supported restoration? A. Thick biotype B. Thin biotype C. Restorative table >6 mm apical to the contact point of the adjacent tooth. D. Positive IL-1 genotype

Answer: A Rationale: A thick biotype provides greater stability for the peri-implant soft tissues with more predictable healing response following surgery and around restorations. Thin biotypes predispose not only to unpredictable healing but also may allow titanium "show through". When implant restorative tables are placed more than 5 mm below the contact point of an adjacent tooth, loss of the papilla is common, having an adverse effect on the final aesthetic result. Finally, genetic testing utilizing a swab has identified that positive intraleuken-1 (IL-1) genotype patients have an increased risk of developing periodontitis with enhanced risk of peri-implant complications. Reference: Buser, Martin and Belser, "Optimizing Aesthetics for Implant Restorations in the Anterior Maxilla: Anatomic and Surgical Considerations", International Journal of Oral and Maxillofacial Implants. Vol. 19, pages 43-61. Tonel, Magner, and Fletcher. "The Effective Distance from the Contact Point to the Crestal Bone on the Presence or Absence of Interproximal Dental Papilla", Journal of Periodontology, 1992. Vol. 63, pages 995-96

91. Which statement is correct regarding pericoronitis of a mandibular 3rd molar? A. Initial treatment can be limited to lavage of material alba and debris B. Pericoronitis occurs only in immunosuppressed patients C. Antibiotics are mandatory D. Extraction should be delayed until completion of a course of antibiotic therapy

Answer: A Rationale: Antibiotics are a key aspect in localizing an infection and limiting its spread to adjacent tissue organs, areas, and spaces. Pericoronal infections that are localized to the immediate enveloping tissues and give no evidence of spread to adjacent tissue planes require local debridement and definitive treatment consisting of removal of the erupting tooth and/or pericoronal tissues. Antibiotics are not mandatory, and tooth extraction need not to be delayed to complete a course of antibiotics. Reference: Ness, GM, Peterson, LJ; Impacted Teeth in Peterson's Principles of Oral and Maxillofacial Surgery, Miloro, M. et al., Editors, Second Edition, BC Decker Inc, 2004, pp140-142.

1. When utilizing a retromandibular approach to a subcondylar fracture, which of the following is true? A. The dissection can go behind or through the parotid gland B. The dissection goes between the temporal and zygomatic branches of the facial nerve C. The approach is best for subcondylar fractures that are high D. The superior extent of the incision begins 2 cm below the earlobe

Answer: A Rationale: Dissections posterior to the parotid and through the parotid have been described. Classically the dissection carried between the cervicofacial trunk and the temporofacial trunk of the facial nerve. The dissection may be on either side of the marginal mandibular branch. This approach is best for lower subcondylar fractures and more limited for higher fractures. The incision begins approximately 0.5 cm below the earlobe. Reference: Hinds EC: Correction of prognathism by subcondylar osteotomy. J Oral Maxillofac Surg 16:209, 1958 Ellis E III, Zide MF: Surgical Approaches to the Facial Skeleton. Baltimore. Lippincott Williams & Wilkins. 1995

57. Which of the following is true regarding submucosal vestibuloplasty? A. A postoperative stent is essential B. A superiosteal dissection is performed C. Split-thickness skin grafts are required D. Commonly used in the mandible

Answer: A Rationale: Distortion/inversion of the upper lip when a mirror is placed to the depth of the maxillary anterior vestibule indicates a lack of vestibular depth (which is why the lip distorts). The Submucous vestibuloplasty is contraindicated in this instance. An open type vestibuloplasty with secondary skin graft or laser vestibuloplasty is a better choice. Reference: Spagnoli, DB, Gollehon, SG, Misiek, DJ; Preprosthetic and Reconstructive Surgery in Peterson's Principles of Oral and Maxillofacial Surgery, Miloro, M. et al., Editors, Second Edition, BC Decker Inc, 2004, pp. 174-175

68. A 23-year-old male is involved in sports related injury and sustains a displacement of his maxillary right central incisor. Which direction of displacement is more likely to lead to pulpal necrosis? A. Intrusion B. Extrusion C. Lateral D. Palatal

Answer: A Rationale: Intrusive forces are more likely to cause direct compressive injury to the pulpal vascular supply than forces in any other direction. The risk of pulpal necrosis after intrusion of teeth with a closed apex is 95% and 65% if the apex is open. Reference: Andreassen JO, Andreassen FM:Essentials of Traumatic Injuries to the Teeth 2nd Edition; Munksgaard, Copenhagen, 2000, pp 77-112.

68. The most important factor associated with the success of osteotome-mediated sinus floor elevation technique is: A. the height of the residual alveolar bone. B. implant design. C. type of graft material used. D. method of sinus in-fracture.

Answer: A Rationale: The ability to obtain primary implant stability is associated with the height of the residual alveolar ridge, and is the primary factor related to success when implants are placed in conjunction with osteotome mediated sinus floor elevation. Implant design graft material and method of sinus in-fracture have minimal influence on survival; however, factors such as osteoporosis, type of final prosthesis, and operator experience may also impact the success of this technique. Reference: Toffler, M., "Osteotome Mediated Sinus Floor Elevation: Clinical Report." International Journal of Oral and Maxillofacial Implants, Vol. 19, No. 2, 2004, pp. 266-273.

91. The minimal bone height required for placement of a transmandibular implant is how many millimeters? A.3 B.6 C.9 D. 12

Answer: A Rationale: The cortical screws for the TMI system are available in lengths of 5,8,11, & 14 mm. Protocol is for the cortical screws to extend beyond the bone for a length of 2mm, but not pierce the periosteum. This allows for tenting of the periosteum and thought to lead to subsequent bone growth. A 3mm mandible is the minimum mandibular height to allow 2mm of "tenting" with the 5mm cortical screw. REFERENCE: Powers MP, Bosker H, Pelt AJW, Dunbar N, " The Transmandibular implant reconstruction system: From progressive bone loss to controlled bone height. JOMS 1994; 52:904-910. Powers MP, Bosker, H The Transmandibular Reconstruction System in Reconstructive and Preprosthetic Surgery, Fonseca and Davis Editors, Second Edition. WB Saunders Co., 1995. pp 565-668.

1. When utilizing the Champy technique for fixation of a mandibular angle fracture, the plate used for fixation is: A. an eccentric dynamic compression plate. B. placed in the zone of compression. C. placed in the zone of neutrality. D. placed in the zone of tension.

Answer: D Rationale: The principle of the Champy technique is for fixation in the zone of tension with natural functional forces aiding in the approximation of the fracture in the area of compression. Reference: Champy M, Lodde JP, Scmitt R, et al. Mandibular osteosynthesis by miniature screwed plates via a buccal approach. Journal of Maxillofacial Surgery 1978; 6:14-9.

36. An afferent pupillary defect in the presence of a normal eye: A. is diagnosed when the pupil reacts to direct stimulation but shows a noticeably more brisk consensual response when the contralateral pupil is directly tested. B. indicates a normal optic nerve in the involved eye. C. has a different response than a Marcus Gunn pupil. D. is associated with pupil asymmetry.

Answer: A Rationale: The diagnosis of an afferent pupillary defect in the presence of normal eye is when the pupil reacts to direct stimulation but shows a noticeably more brisk consensual response when the contralateral pupil is directly tested. It indicates an optic nerve injury. An afferent pupillary defect is also call a Marcus Gunn pupil. Pupillary symmetry is not affected by the afferent pupil defect. This implies an efferent lesion or an iris lesion or both. Reference: Gossman MD, Roberts DM, Barr CC, Ophthalmic aspects of orbital injury; A comprehensive diagnostic and management approach. Clin Plastic Surg 1992; 19:17-85

57. The difference between infected human and animal bites is the presence of: A. pasteurella multocida. B. staphylococcus aureus. C. eikenella corrodens. D. fusobacterium nucleatum.

Answer: A Rationale: The human and animal population is very similar in their oral flora. Each has a predomination of Streptococcus species and numerous gram negative anaerobic organisms. Each has numerous species of viruses and some mycobacteria have been reported. It is however, Pasteurella multocida that is only found in the non-human population Reference: Haug RH, Assael L, "Infections in the Maxillofacial Trauma Patient", Chapter 17 in Topazian RG, Goldberg MH, Hupp J (Eds.). Oral and Maxillofacial Infections. Philadelphia PA, W B Saunders, 2002, pp 359-380.

36. Failure after replantation of an avulsed tooth occurs mostly due to: A. internal resorption, external resorption, pulpal necrosis. B. external resorption, ankylosis, periodontal disease. C. internal resorption, tooth migration, ankylosis. D. external resorption, pulpal necrosis, periodontal disease.

Answer: A Rationale: The most common tooth avulsed is the maxillary central incisor. Treatment is geared towards early re-establishment of the PDL cell physiology, via solutions with physiologic pH and osmolarity. Re-implant and stabilize the tooth within two hours of avulsion. Reference: Abubaker AO, Giglio JA, Mourino AP, Diagnosis and management of dentoalveolar injuries. In:Fonseca RJ:Oral and Maxillofacial Surgery, Vol 1. W.B. Saunders Co, Philadelphia, 2000. Miloro M, et al. Peterson's Principles of Oral and Maxillofacial Surgery, 2nd Edition. Pg 393-395. BC Decker Inc, 2004.

36. Irreversible ischemic damage to the retinal tissue secondary to pressure and hypoxia has been shown: A. within 60 minutes to 2 hours following a traumatic event. B. avoidable by use of megadose steroids which should be initiated 48 hours after injury. C. to respond better to treatment when the patient has no initial light perception. D. to respond better when there is penetrating trauma versus blunt trauma.

Answer: A Rationale: The optic nerve and retinal tissue, unlike skeletal muscle, have been shown to be extremely sensitive to pressure and hypoxia. Irreversible ischemic damage often occurs within 60 minutes and certainly within 2 hours. Megadose steroids have been recommended during the first 8 hours following injury. The outcome in patients with traumatic optic neuropathy with no light perception on presentation is a poor prognostic factor. Likewise patient presenting with traumatic optic neuropathy after penetrating trauma have a worse prognosis than those with blunt facial trauma. Reference: Wang BH, Robertson BC, Girotto JA, Liem A, Miller NR, Iliff N, Manson P, Traumatic optic neuropathy: A review of 61 patients. Plastic Reconstructive Surg 2001; 107:1653-1664

91. In 75% of patients, in order for the dental papilla to fill the embrasure space of a single tooth dental implant supported restoration, the maximum distance between the crestal bone and the contact point is: A. 6mm. B. 5mm. C. 4mm. D. 3mm.

Answer: A Rationale: The presence or absence of a peri-implant papilla mainly depends on the distance between the alveolar crest and the contact point. In single-tooth gaps, the bone height at adjacent teeth determines the status of the papilla. A clinical study by Dennis Tarnow , et al, demonstrated that a distance of 6mm or more from the alveolar crest to the contact point reduces the probability of intact papillae. Reference: Tarnow DP, Magner AW, Fletcher P. The effect of the distance from the contact point to the crest of bone on the presence or absence of the interproximal dental papilla. J Periodontology 1992;63:995-996

36. A comminuted angle fracture of the mandible would be treated most effectively with the followingfixation: A. ORIF with reconstruction bone plate applied to the inferior border with 3 bone screws on each side of the fracture. B. antibiotics and soft diet for 4 weeks. C. anatomic reduction with Champy bone plate at the superior border. D. maxillo-mandibular fixation to restore occlusion for 4 weeks.

Answer: A Rationale: The reconstruction plate is designed to bear the entire load of the mandibular movement across the fracture site. The remaining methods mentioned do not adequately stabilize the fracture segments and increase the possibilities for non-union and infection. Reference: Miloro, M. et al, Peterson's Principles of Oral and Maxillofacial Surgery, 2nd Edition, page 375, BC Decker INC, 2004

57. The differences between the child and adult respiratory apparatus are that children possess: A. greater diaphragmatic breathing B. high lung compliance. C. ribs and sternum that are rigid. D. a low metabolic rate.

Answer: A Rationale: The respiratory apparatus and physiology in children differs from adults in many ways. Children have a high metabolic rate, thus a high oxygen demand and hypoxemia develops rapidly their upper airway has a smaller caliber, large soft tissues, larynx that is positioned cephalad, with a shorter trachea, and epiglottis that is short and narrow. This results in increased airway resistance, easy obstruction, difficult intubation, easy extubation. The child's ribs and sternum are compliant, and when respiratory efforts are diminished, their reserve decreases. The child relies more on diaphragatic breathing, therefore high intrathoracic pressure or abdominal distention diminishes ventilation. The child possesses low lung compliance and therefore ventilation is inefficient during respiratory distress. Reference: American College of Surgeons Committee on Trauma:ATLS Advanced Trauma Life Support Program for Doctors. 7th Edition, American College of Surgeons, Chicago, IL, 2004, pgs 243-262.

68. Which of the following is more characteristic of a thin split-thickness skin graft (STSG) when compared to a thick STSG? A. Increased probability of graft survival B. Decreased secondary contracture of the graft C. More likely to result in recipient site hair growth D. Slower donor site re-epithelialization

Answer: A Rationale: The thin STSG is more likely to survive on its recipient site because it can survive well during the phase of plasmatic absorption and therefore wait longer for vascularization. The thicker the graft, the less its tendency to undergo secondary contraction; full-thickness skin grafts show little or no evidence of contracture. The thicker the graft, the more likely a hair follicle will be transplanted. The thinner a graft, the more accessory skin structures remain at the donor site for epithelial growth. Reference: Fonseca, Davis. Reconstructive Preprosthetic Oral & Maxillofacial Surgery (1st Edition). Page 35, W.B Saunders Company, 1986 Philadelphia

1. Penetrating trauma located just above the clavicles would be an injury to what zone of the neck? A. Zone 1 B. Zone 2 C. Zone 3 D. Zone 4

Answer: A Rationale: The zones of the neck were originally described by Dr. Monson in 1969. Zone 1 is defined as the area from the clavicles to the cricoid cartilage. The risk of injury to the great vessels is common in this area. Zone 2 represents the area from the cricoid cartilage to the angle of the mandible. It is the largest area of the neck and thus is most likely to be injured with penetrating neck trauma. Zone 3represents the region from the angle of the mandible to the base of the skull. The area presents the most difficult area for surgical access. Reference: Reference:Monson DO, Saletta JD, Freeark RJ. Carotid Vertebral Trauma. J Trauma 9:987- 99, 1969.

68. The above axial CT scans are from a patient who developed instantaneous facial swelling during surgical extraction of an erupted tooth utilizing a high-speed dental handpiece. The scans were taken immediately following the procedure. What is the most appropriate management? A. An antibiotic course and observation B. Hyperbaric oxygen therapy C. A cervical drainage procedure D. A mediastinal drainage procedure

Answer: A Rationale: These are axial CT scans of air emphysema. The use of a high-speed dental handpiece for the procedure and the instantaneous swelling are characteristic of this phenomenon. The air was introduced by the handpiece into the cheek, and has progressed along fascial planes into the carotid sheath and then the mediastinum. The treatment of tissue emphysema varies with the severity of the condition. Most cases of subcutaneous emphysema will begin to resolve after 2 to 3 days of supportive treatment, and residual swelling is usually minimal after 7 to 10 days of observation. Treatment is usually conservative, and consists of antibiotic coverage to prevent infection. Oral bacteria may possibly be carried with the aerosol into the soft tissue and represent a potential nidus of infection. Additionally, a course of systemic corticosteroids may promote faster resolution. Surgical decompression of the extensive emphysema should not be routinely used, because it is likely to be ineffective and may even worsen or spread the emphysema. In most cases the patient can be discharged with an explanation of the nature of the tissue emphysema and its course. Reference: Peterson LJ. Emphysema and dental drill (comment). J Am Dent Assoc 1990; 120:423 Pynn BR, Amato D, Walker DA. Subcutaneous emphysema following dental treatment:report of two cases and review of the literature. J Can Dent Assoc 1192; 58:496-499 Schuman NJ, Edwards BC, Walker W. Subcutaneous emphysema during operative dentistry:report of a case with a thirty-month follow up. J Oral Med 1983; 38:168-169 Woehrlen AE. Subcutaneous emphysema. Anesth Prog 1985; 32:161-163 Salib RJ, Valentine P. Surgical emphysema following dental treatment. The Journal of Laryngoscopy and Otology 1999; 113:756-758

1. A 21-year-old female is an unrestrained driver involved in a MVA. She suffers a scalp laceration and is noted to have lost 1000mL of blood at the scene. You would expect her vital signs to be consistent with: A. Pulse rate >100, normal systolic blood pressure, decreased pulse pressure, respiratory rate of 20-30, urinary output of 20-30mL/hr. B. Pulse rate <100, normal systolic blood pressure, normal or increased pulse pressure, respiratory rate of 14-20, urinary output of >30mL/hr. C. Pulse rate >120, decreased systolic blood pressure, decreased pulse pressure, respiratory rate of 30-40, urinary output of 5-15mL/hr. D. Pulse rate >140, decreased systolic blood pressure, decreased pulse pressure, respiratory rate of >35, urinary output that's negligible.

Answer: A Rationale: These findings are consistent with a Class II hemorrhage, 750-1500ml, The vitals signs or such a blood loss are consistent with those in response A. Response D reflects the vital signs of a Type IV blood loss, Response C a Type III and Response B a Type I. Reference: 1997 ATLS for Doctors, Sixth Edition.

68. Which of the following can cause premature loss of temporary implants used for the purpose of providing orthodontic anchorage? A. Non-keratinized surrounding mucosa B. Type of intended tooth movement C. Mini-plate implant configuration D. Mini-screw implant configuration

Answer: A Rationale: While success rates for mini-implants are high, studies like that of Chang and Tseng demonstrate that implants placed through non-keratinized mucosa have a higher rate of failure. Another factor that may have an adverse affect on success is placement in the posterior mandible. Implant type (mini-screw of mini-plate) does not effect failure rates of rigid orthodontic anchorage. Reference: Chang, S and Tseng, I, International Journal of Oral and Maxillofacial Implants, 2004, Vol. 19, pages 100-106,

53. β2 transferrin is found only in: A. cerebrospinal fluid, aqueous humor and perilymph. B. cerebrospinal fluid, serum and aqueous humor. C. cerebrospinal fluid, nasal secretions and perilymph. D. serum, nasal secretions and aqueous humor.

Answer: A Rationale: β2 transferrin is one of two major variants of iron binding transferrin glycoproteins forum in several forms of various body fluids. β2 transferrin lacks a carbohydrate side chain which increases its positive charge, permitting isolation during electrophoresis. β2 transferrin is only found in cerebrospinal fluid, aqueous humor and perilymph. Thus, with the exception of contamination by a ruptured globe, it is an ideal substance to distinguish between cerebrospinal fluid, nasal secretions and serum. Reference: Brandt MT, Jenkins WS, Fattahi TT, Haug RH:Cerebrospinal Fluid:Implications in Oral and Maxillofacial Surgery. J Oral Maxillofac Surg. 60:1049-1056, 2002.

91. When performing immediate placement of an implant into extraction site of a maxillary central incisor, which of the following statements is true? A. The tooth socket must be completely obliterated by the dental implant. B. Immediate placement is not recommended if buccal plate integrity is lost. C. The thick flat gingival biotype has a higher risk of buccal plate resorption. D. The implant should be placed adjacent to the buccal wall of the extraction site.

Answer: B Reference: Hämmerle CHF, et al, Consensus Statements and Recommended Clinical Procedures Regarding the Placement of Implants in Extraction Sockets, Int. J of Oral & Maxillofacial Implants, Vol 19 Supp, pp 26-28

31. Which of the following surgical approaches to the inferior orbital rim has the least potential for producing post-operative ectropion and poor cosmesis? A. Infraorbital B. Subciliary C. Transconjunctival D. Inferior Lid

Answer: C Rationale: Infraorbital or rim incision results in the worst esthetics and offers no advantages. The subciliary incision has an ectropion rate of approximately 6 %, whereas the transconjunctival incision has an ectropion/entropion rate of approximately 1.2 %. Thus, the transconjunctival incision as the approach of choice to avoid ectropion. Reference: Miloro M, et al. Peterson's Principles of Oral and Maxillofacial Surgery, 2nd Edition. BC Decker Inc, 2004.

22. A 9-year-old child is brought to the emergency room after being struck by a car. The patient is unresponsive to command and breathing only infrequently. Cervical immobilization is in place. Oral intubation attempts are unsuccessful because of brisk bleeding from facial fractures. The most appropriate next step is: A. obtain an angiogram B. surgical cricothyroidotomy. C. surgical tracheostomy. D. percutaneous tracheostomy.

Answer: B & C Rationale: The child is too young for cricothyroidotomy and percutaneous tracheostomy is not indicated for emergency airway. Cricothyroidotomy is not recommended in children less than 12 years of age, since the cricoid cartilage is the only circumferential support to the upper trachea. ATLS teaches that a child may be temporarily oxygenated with needle jet insufflation, but this technique does not provide ventilation and is not a definitive airway. Although surgical tracheostomy is unappealing in this age category, it is the procedure of last resort in this scenario. Reference: Advanced Trauma Life Support for Doctors Course Student Manual, 7th Ed., First Impression Press, 2004, page 48,49,250

68. If the lingual alveolar plate is fractured and mobile during removal of an erupted mandibular third molar, the fractured segment should be: A. stabilized to avoid damage to the lingual nerve. B. left in place with minimal manipulation. C. removed with careful subperiosteal dissection. D. removed and the lingual nerve explored for evidence of injury.

Answer: B Rational: Minimal manipulation of the segment will provide the greatest chance of maintaining the periosteal attachment, and thus the blood supply to the segment. If soft tissue is inadvertently dissected from the alveolus, the segment of bone will likely undergo necrosis. Removal of the segment is generally not advised, dissection in this region would increase the risk of injury to the lingual nerve. REFERENCE: Berman SA. Basic principles of dentoalveolar surgery. In:=Petersen LJ, Indresano AT, Marciani RD, Roser SM. Principles if Oral and Maxillofacial Surgery. Philadelphia, PA; JB Liincott CO; 1992, pp. 95-96. Peterson, LJ; Prevention and Management of Complications in Peterson, Ellis, Hupp, Tucker. Contemporary Oral & Maxillofacial Surgery 4th edition. pp 228-230., Mosby, 2003 USA. Peterson LJ:Contemporary Oral and Maxillofacial Surgery, 3rd edition Ed. Pg. 261 - 262. Mosby, Missouri

36. A CT scan reveals an orbital fracture with a trap door appearance with clinical and radiographic eidence of inferior rectus muscle incarceration is: A. more likely to occur in adults than in children. B. warrants early intervention to free up the tissues. C. can be managed by steroids and delay of surgery for 3 weeks D. frequently requires the use of an orbital implant to bridge the floor defect.

Answer: B Rationale: A CT scan of an orbit that has a trapdoor appearance with clinical and radiographic evidence of inferior rectus muscle incarceration is more like to occur in children than in adults and it warrants early intervention to prevent ischemic necrosis to the rectus muscle. Steroids are may be used in cases where there is no evidence of muscle entrapment and mild restricted motility. By its nature, a trapdoor defect seldom requires an implant to bridge a defect. Reference: Ochs MW, Orbital and ocular trauma. In: Miloro M ed, Peterson's Principles of Oral and Maxillofacial Surgery Second edition BC Deker, London 2004 Chapter 24 pp 479

1. A 65-year-old man fell down the stairs. Upon examination of him, you notice that he opens his eyes to speech, localizes pain, and mutters inappropriate words. You assess his Glasgow coma scale (GCS) to be: A. 13 B. 11 C. 9 D. 7

Answer: B Rationale: According to the Glascow Coma Scale, the patient can open his eyes in response to commands speech, (3 out of 4); localizes pain, (5 out of 5); yet produces inappropriate words, (3 out of 6); for a Glascow coma score of 11. Reference: 1997 ATLS for Doctors, Sixth Edition

68. Which of the following is a characteristic of cracked tooth syndrome? A. Percussion sensitivity B. Pain on release of bite force C. More likely to occur in a tooth without restorations. D. The most appropriate treatment is extraction.

Answer: B Rationale: Cracked tooth syndrome frequently occurs in teeth that have had 1 or more restorations in the marginal ridge area. The tooth frequently exhibits pain on release of bite force, rather than on percussion. Restoration with onlays, composite resin, or splinting with an orthodontic band is frequently successful. Extraction is necessary only if the tooth root fractures vertically. Reference: Homewood CI. Cracked tooth syndrome--incidence, clinical findings and treatment. Australian Dental Journal. 43(4):=217-22, 1998 Aug.

68. During extraction of teeth #14 and #15, the two teeth were delivered with a large attached piece of the buccal alveolar plate. An 8 mm diameter oro-antral communication is noted. The most appropriate treatment courseis: A. attempt suturing as best as possible with available local tissue. B. develop a buccal mucoperiosteal flap and advance it to achieve primary closure without tension. C. develop a free palatal mucosa graft to passively cover the defect. D. immediately proceed with packing the sinus with iodoform gauze saturated with triple antibiotics and perform a nasal antrostomy.

Answer: B Rationale: Acute oroantral communication is a common occurrence during extraction of the posterior maxillary teeth. In most instances these perforations are small and the blood clot that fills the extraction site usually seals such small perforations. If there is an infection in the antrum, the defect is large (>5 mm in diameter), the gingival tissue are not approximated, the wound is dehisced, or the patient does not follow the postoperative instructions, an oro-antral fistula occurs. Although option A seems reasonable for a small fistula, due to the size of the defect, local suturing without flap development will most likely not result in primary closure, and then result in an oro-antral communication. Option B offers the best chance of primary closure of the large communication and prevents development of de novo sinusitis. A free graft cannot survive over a hole without its own blood supply and thus option C is incorrect. Option D is also incorrect because it treats chronic sinusitis. There is no indication in the stem of the question that sinusitis occurred. Closure of the communication is not addressed. Reference: Laskin, D.M.:Management of Oro-antral fistula and other sinus-related complications. In:Oral Maxillofac Surg Clinic North America Vol. Number 1, Feb. 1999, W.B. Saunders, Philadelphia, pp 155-179.

68. The biological width surrounding natural teeth and freestanding osseointegrated root form implants demonstrate a relatively constant thickness of: A. 1mm. B. 3 mm. C. 5 mm. D. 7 mm.

Answer: B Rationale: An appreciation of biologic width is important in best determining implant positioning to allow for an aesthetic result. Numerous studies document the dimensions of biological width for both natural teeth and implants. Biologic width is comprised of the zone of supracrestal connective tissue that measures approximately 1 mm and epithelial structures, including the junctional and sulcular epithelium that measure about 2 mm in height. Reference: Burglund H, T. and Lindhe, J. "Dimensions of the Peri-Implant Mucosa; Biological Width Revisited". Journal of Clinical Periodontology, 1996. Vol. 23, pages 971-973. Herman, J. S, Buser, D., Shenk, R.K., Higginbottom, F.L., and Cochran, D. L. "Biological Width Around Titanium Implants:A Physiologically Formed and Stable Dimension Over Time". Clinical Implants Res., 2000. Vol. 11, pages 1-11.

57. The circulatory system for children in trauma differs from adults in that children have a proportionally: A. higher cardiac output. B. smaller stroke volume. C. equal blood volume. D. decreased physiologic reserve.

Answer: B Rationale: Children possess a higher cardiac output than adults. Under the physiologic stress of trauma, the child's stroke volume cannot increase, so that under stress proportionate increases in stroke volume cannot occur and increases in cardiac output are rate determined. Bradycardia could result in hypoxia and hypercapnia, and is ominous. The child's proportional blood volume is greater than an adult. However, the absolute blood volume and therefore tolerable absolute blood loss is less, so that minor injuries may result in blood loss which is physiologically significant. The child's blood pressure is maintained through physiologic compensation (vasoconstriction, tachycardia, myocardial contractility). This compensation may be misleading by masking volume reduction. The healthy child has an increased total systemic physiologic reserve, thus with trauma, symptoms of hypoxemia or hypovolemia may not occur until a potentially catastrophic hypovolemia has occurred. Reference: American College of Surgeons Committee on Trauma:ATLS Advanced Trauma Life Support Program for Doctors. 7th Edition, American College of Surgeons, Chicago, IL, 2004, pgs 243-262.

1. When treating uncomplicated, compound mandible fractures, the current recommendation for antibiotics isthat: A. antibiotic coverage is not needed. B. perioperative antibiotic use is indicated. C. pre-op antibiotics with a 7 day post-operative course are indicated. D. antibiotics are only useful for open reductions.

Answer: B Rationale: Compound or open mandible fractures have been shown to have reduced infection rates when perioperative antibiotics have been used. The use of postoperative courses of antibiotics has not been shown to affect the rate of infection. Reference: Zallen RD, Curry JT:A study of antibiotic usage in compound mandibular fractures. J Oral Surg; 33:431, 1975. Abubaker AO, Rollert MK:Postoperative antibiotic prophylaxis in mandibular fractures:A preliminary randomized double-blinded and placebo controlled clinical study. J Oral Maxillofac Surg; 59:1415, 2001.

68. Which of the following is an absolute contraindication for dental implant placement? A. Oral lichen planus B. Uncontrolled periodontal disease C. Insulin dependent diabetes mellitus D. Cigarette smoking

Answer: B Rationale: Contraindications for the use of an implant restoration are in developing patients (particularly in the maxilla, where vertical growth continues after permanent teeth are fully erupted), uncontrolled periodontal disease, aesthetic areas with thin, highly scalloped gingiva, adjacent periapical pathology, and non motivated patients. Relative contraindications where adjacent root flaring precludes placement (correction needed with orthodontics), smokers (increased failure rate especially in type IV bone), connective tissue diseases, and diabetes and autoimmune diseases. Reference: Salinas TJ, Block MS, Sadan A. Fixed partial denture or single-tooth implant restoration? Statistical considerations for sequencing and treatment. J Oral Maxillofac Surg. 2004 Sep;62(9 Suppl 2):2-16. Lindquist LW, Carlsson GE, Jemt T. Association between marginal bone loss around osseointegrated mandibular implants and smoking habits:A 10 year follow- up study J Dent Res 1997;76:1667. De Bruyn H, Collaert B. The effect of smoking on early implant failure Clin Oral Implants Res 1994;5:=260 Isidor F, Brondum K, Hansen HJ. Outcome of treatment with implant-retained dental prostheses in patients with Sjögren syndrome Int J 1999;14:736. Balshi TJ, Wolfinger GJ. Dental implants in the diabetic patient:A retrospective study Implant Dent 1999;8:355 Rajnay ZW, Hochstetter RL. Immediate placement of an endosseous root-form implant in an HIV-positive patient:Report of a case J Periodontol 1998;69:=1167

36. A 5cm. full thickness scalp avulsion down to the cranium is best treated by a: A. split thickness skin graft. B. full thickness skin graft. C. split thickness skin graft placed over bur holes made in the outer table. D. rotation-advancement flap.

Answer: D Rationale: Exposed cortical cranial bone will not accept skin grafts. Rotational-advancement flaps with multiple galeal releasing incisions parallel to the long axis of the flap will provide adequate coverage in most instances. Reference: Fonseca RJ and Walker RV, Management of soft tissue injuries, Oral and Maxillofacial Trauma, vol 1, WB Saunders, 2001

57. For alveolar distraction to be successful to increase alveolar ridge height and width it is imperative that the: A. transport segment not be stabilized by screws to prevent resorption. B. periosteum be maintained on the crestal alveolus of the transport segment. C. transport segment be increased in size due to resorption during the process. D. process be completed as quickly as possible.

Answer: B Rationale: Distraction of the alveolus is successful only if the periosteal blood supply is maintained to the transport segment. The segment must be secured to the device to keep it stable during the process by rigid fixation. Minimal resorption of the transport segment occurs due to the intact periosteal blood supply. The process must be governed by the latency, activation, and consolidation phases and not hurried Reference: Spagnoli, DB, Gollehon, SG, Misiek, DJ; Preprosthetic and Reconstructive Surgery in Peterson's Principles of Oral and Maxillofacial Surgery, Miloro, M. et al., Editors, Second Edition, BC Decker Inc, 2004, pp183-4

53. In the recumbent position, normal intracranial pressure is: A. 1 to 5 mm Hg. B. 8 to 12 mm Hg. C. 15 to 19 mm Hg. D. 22 to 26 mm Hg.

Answer: B Rationale: In the recumbent position, intracranial pressure, and thus cerebral spinal fluid pressure, is about 8 to 12 mm Hg or 110 to 150 mm H2O. Autoregulation maintains this pressure. Reference: Brandt MT, Jenkins WS, Fattahi TT, Haug RH:Cerebrospinal Fluid:Implications in Oral and Maxillofacial Surgery. J Oral Maxillofac Surg. 60:1049-1056, 2002.

31. An injury to the temporo-facial trunk of the facial nerve is suspected within a deep cheek laceration. Which of the following will best confirm an intact and functioning facial nerve? A. A normal Bell's phenomenon B. An intact corneal reflex C. Absence of divergent strabismus D. Absence of an oculo-cardiac reflex

Answer: B Rationale: Integrity of the temporo-facial trunk of the facial nerve can be assessed by performing a corneal reflex (tearing to touching the ipsilateral cornea.) The afferent and efferent limbs of the reflex involve the first division of the trigeminal nerve and facial nerve respectively. Strabismus is associated with defects in cranial nerves III, IV, and VI. Reference: Bates, B, A Guide to Physical Examination and History Taking, The Nervous System, JB Lippincott 2002

36. Success after replantation of an avulsed tooth that was preserved in a clean and moist container by the patient is most dependent on: A. sterilization before replantation. B. replantation within 30 minutes after avulsion. C. curettage of the tooth root and socket. D. rigidity of fixation splinting.

Answer: B Rationale: Optimal success of treatment is to replant and stabilize avulsed teeth within 2 hours. The PDL cells become irreversibly necrotic after this time frame. Treatment with a nonrigid splint will allow movement of the tooth, allowing the PDL to heal. Reference: Abubaker AO, Giglio JA, Mourino AP, Diagnosis and management of dentoalveolar injuries. In:Fonseca RJ:Oral and Maxillofacial Surgery, Vol 1. W.B. Saunders Co, Philadelphia, 2000. Miloro M, et al. Peterson's Principles of Oral and Maxillofacial Surgery, 2nd Edition. Pg 394-5. BC Decker Inc, 2004.

36. Generally speaking, soft tissue injuries associated with dentoalveolar trauma are treated: A. before management of all hard tissue injuries. B. after management of all hard tissues injuries. C. before management of bony injuries, but after management of dental injuries. D. should never be closed primarily but should be allowed to heal by secondary intention.

Answer: B Rationale: Soft tissue wounds associated with dentoalveolar trauma are always treated after management of the hard tissue. This prevents wasting time of suturing wounds that are likely to be removed or compromised during the intraoral manipulation to treat the hard tissue or dental injuries. Reference: Peterson LJ, Contemporary Oral and Maxillofacial Surgery, 2nd edition, Ed. Pg. 583. JB Lippincott Co., Philadelphia 1992

91. What stage of root development is optimal for the transplantation of a developing mandibular third molar to a first molar extraction site? A. Less than 1/2 B. Between 1/2 and 2/3 C. Between 2/3 and complete D. Complete

Answer: B Rationale: The donor third molar roots should be no less than half and preferably about two thirds developed. Reference: Silva, RG, Pogrel, MA; Tooth Reimplantation and Transplantation in Fonseca, RJ, Oral and Maxillofacial Surgery Vol 1, Anesthesia/Dentoalveolar Surgery/ Office Management, WB Saunders 2000. pp 299-301. Kaban, LB, Dentoalveolar Surgery in Kaban Pediatric Oral and Maxillofacial Surgery, WB Saunders, 1990, p109.

68. During extraction of erupted tooth #19, the mesial root tip fractures. During your attempt to elevate, the fragment disappears through the socket. The most appropriate surgical approach to retrieve the root tip is: A. create a buccal flap and remove the buccal alveolar plate to gain access. B. develop a lingual flap. C. a Risdon approach. D. widen the cortical defect through which the root disappeared.

Answer: B Rationale: The mylohyoid muscle forms the floor of the mouth attaching to the mylohyoid ridge. The latter curves superiorly as it approaches the third molar. Roots of first molars are usually superior to level of mylohyoid muscle attachment to the mandible. The root tips of third molars are usually inferior to the muscle attachment. Also, the lingual plate is thin in this site. Therefore, option B is correct. Options A and D are both incorrect since extensive procedures that may not gain visualization of the displaced root, are required. Option C is incorrect because anatomically the root is likely to be superior to mylohyoid. If a third molar root was displaced, it may be inferior to the mylohyoid. In this latter situation the submandibular approach may be necessary. Reference: Gregg, J.M.:Surgical Anatomy. In Oral and Maxillofacial Surgery, Vol. 1, edit:D.M. Laskin. The C.V. Mosby Company, St. Louis, 1980, pp 3-49. Pogrel, M.A.:Complications of Third Molar Surgery. In Oral and Maxillofacial Surgery Clinics of North America Vol 2, Number 3, 1990, W.B. Saunders Company. Pp. 441-451.

68. Which erupted maxillary tooth is usually associated with the highest rate of oral-antral communication after routine extraction? A. Maxillary second premolar B. Maxillary first molar C. Maxillary second molar D. Maxillary third molar

Answer: B Rationale: The palatal root of the maxillary first molar is usually the most closely related to the floor of the maxillary sinus. Reference: Alling CC (Ed). Dentoalveolar Surgery. Oral and Maxillofacial Surgery Clinics of North America. Vol 5. WB Saunders. Philadelphia, 1993

1. Alignment of which of the following is the most reliable for proper reduction of the zygomaticomaxillary complex fracture? A. Frontozygomatic suture B. Sphenozygomatic suture C. Infraorbital rim D. Medial orbital rim

Answer: B Rationale: The sphenozygomatic suture area has been previously analyzed and shown to be an area for confirmation of alignment of the zygomatic arch and the zygomatic complex (ZMC). This has also been shown to key point for fixation thru biomechanical studies. The sphenozygomatic suture is a broad area along the greater wing of the sphenoid and can be approached along the internal aspect of the lateral orbit. Even in severe midface fractures the greater wing of the sphenoid is intact thus acting as a key landmark for proper reduction of the ZMC fracture. Reduction of the frontozygomatic suture or the infraorbital rim alone can result in errors due to the small surface area. The medial orbit is generally not involved in a ZMC fracture. Reference: Rohner D, Tay A, Meny CS, Hutmacker DW, Hammer B.: The sphenozygomatic suture as a key site for osteosynthesis of the orbitozygomatic complex in panfacial fractures: A biomechanical study in human cadavers based on clinical practice. Plast Reconstr Surg 110: 1463, 2002. Manson PN, Clark N, Robertson B, et al. Subunit principles in midface fractures: the importance of sagittal buttresses, soft tissue reductions and sequencing treatments of segmental fractures. Plast Reconstr Surg 103: 1287, 1999.

91. Which of the following statements regarding adjunctive medications administered to an 18-year- old healthy male during removal of 3rd molars, is true? A. Prophylactic antibiotics are recommended to prevent post-operative infection. B. Local measures are as effective as systemic antibiotics in the reduction of postoperative dry sockets. C. Perioperative steroids administered to reduce post-operative swelling have no appreciable affect on dry socket incidence D. Maximum control of swelling requires no additional steroid doses other than the initial perioperative dose

Answer: B Rationale: The usage of prophylactic antibiotics involves the issue of risk versus benefits. Without further extenuating conditions, the increase of antibiotic-related complications, e.g. allergy, resistant bacteria, gastrointestinal side effects, and secondary infections, is not outweighed by the benefits in an otherwise uncomplicated 3rd molar surgery. Perioperative steroids do have a major clinical impact on swelling in the early postoperative period but, for maximum control, additional steroids must be given for 1 or 2 days following surgery. The administration of perioperative steroids may increase the incidence of alveolar osteitis after third molar surgery, the data are lacking as to the precise degree of increase. Reference: Ness, GM, Peterson, LJ; Impacted Teeth in Peterson's Principles of Oral and Maxillofacial Surgery, Miloro, M. et al., Editors, Second Edition, BC Decker Inc, 2004, pp 147-149.

36. Calcium phosphate bone cement used for pediatric frontal sinus obliteration: A. is an almost ideal alloplast for this application. B. breaks up when placed directly over dura or sinus membranes. C. is not replaced by bone and remains inert. D. is readily available and inexpensive.

Answer: B Rationale: While calcium phosphate bone cement is a near ideal alloplast in that it has unlimited availability, is biocompatible, has high mechanical strength, has a low risk for infection, is radiopaque and is incorporated into the bone, it does have drawbacks. It is expensive, and when placed directly over dura or sinus membranes, it breaks up and a chronic foreign body reaction ensues. This is particularly true for the pediatric population. Reference: D'Addario M, Haug RH, Talwar R: Biomaterials for Use in Frontal Sinus Obliteration. Journal of Long-Term Effects of Medical Implants. 14: 455-465, 2004.

36. After surgical treatment of a zygomaticomaxillary complex fracture, the immediate treatment of a retrobulbar hematoma accompanied by loss of visual acuity and increased intraocular pressure is: A. decompression of the frontal sinus. B. intravenous corticosteroids. C. lateral canthotomy. D. medial canthotomy.

Answer: C RATIONALE: The emergency procedure of choice for acute visual acuity loss associated with acute orbital compartment syndrome is dissection of the lateral canthus and disinsertion of at least the inferior crus of the lateral canthal tendon, which allows complete mobility of the lower lid. Visual loss without clear signs consistent with increased IOP is not an indication for this procedure. Other primary indications for lateral canthotomy and cantholysis include an IOP greater than 40 mm Hg and proptosis, which may be used as a criterion for unconscious patients whose visual acuity cannot be determined. Secondary criteria include afferent pupillary defect, ophthalmoplegia, cherry-red macula, optic nerve head pallor, and severe pain, but these are all considered less sensitive or very late signs. Contraindication for this procedure would be a suspected ruptured globe. REFERENCE: Fonseca, RJ, Walker, R, Oral and Maxillofacial Trauma Volume 1, WB Saunders, 1991

36. The best technique to manage a 40% traumatic avulsion of the upper eyelid is: A. allow the wound to heal by secondary granulation. B. undermine and close primarily. C. split thickness skin graft from the ipsilateral postauricular region. D. full thickness skin graft harvested via blepharoplasty approach from the contralateral upper eyelid.

Answer: D Rationale: Full thickness skin graft from the opposite upper lid is preferred due to similar skin thickness, color, texture and availability with minimal donor site morbidity. This avoids scar and ectropion. Reference: Sandler NA, Evaluation and management of traumatic eyelid injury, OMS Knowledge Update vol 3, AAOMS, 2001

36. To restore proper projection of facial width and orbital volume in a patient with a high energy grossly comminuted zygomaticomaxillary fracture one should: A. use a Carroll- Girard screw placed transcutaneous to reduce the fracture in a closed method. B. align the comminuted segments of the infraorbital rim to assure proper placement of the zygoma. C. expose the sphenozygomatic suture to aid in anatomic reduction. D. expose and plate the zygomatic-frontal suture area before exposing other areas of the zygoma.

Answer: C Rationale: A grossly comminuted zygomatic complex fracture will not be stable with a closed technique. Comminuted segments of the infraorbital rim are a poor guide to placement of the zygoma. The zygomatic-frontal suture is a poor reference for placement of the zygoma. In high energy grossly comminute fractures, the sphenozygomatic suture with its broad contact can aid in the anatomic reduction of the fracture. Reference: Baley JS, Goldwasser MS, Management of Zygomatic complex fractures. In: Miloro M ed, Peterson's Principles of Oral and Maxillofacial Surgery Second edition BC Deker, London 2004 Chapter 23.2 pp 452

68. The angiogenic effects of hyperbaric oxygen therapy occur at regions of: A. tissue hypoxia. B. a shallow oxygen gradient C. a steep oxygen gradient D. tissue hyperoxia

Answer: C Rationale: HBO creates an oxygen gradient in radiated tissue which does not normally exist. This steep oxygen gradient stimulates the normal wound healing mechanism that is missing in radiated tissue. The effects of HBO fall off when the tissue is sufficiently revascularized so that it no longer generates a sufficient oxygen gradient to continue the angiogenic process. Reference: Fonseca, Davis. Reconstructive Preprosthetic Oral & Maxillofacial Surgery (2nd Edition). Page 1107-08, W.B Saunders Company, 1995 Philadelphia.

57. Which of the following statements is true regarding alveolar distraction? A. The consolidation period is the time after the osteotomy and prior to distraction B. Activation occurs 3mm/day in 3 divided, equal segments per day C. The consolidation phase should be three times the length of the active distraction period D. The latency period should be shortened for distraction of irradiated bone

Answer: C Rationale: After the osteotomy is performed and the distraction device placed, a latency period must be observed. The latency period can be from 4-7 days depending on age of patient, blood supply, irradiation to the area, scar tissue, etc. Secondary to impaired blood supply, the latency period should be longer in irradiated bone. The latency period is followed by the active distraction period which varies depending on the transport distance. The rate and rhythm of distraction is how far and how often. A rate of 1mm/day at a rhythm of .25mm four times a day is ideal. The consolidation period begins when active distraction stops and is generally three times the active distraction period. Reference: Spagnoli, DB, Gollehon, SG, Misiek, DJ; Preprosthetic and Reconstructive Surgery in Peterson's Principles of Oral and Maxillofacial Surgery, Miloro, M. et al., Editors, Second Edition, BC Decker Inc, 2004, pp184-5

91. Which of the following statements, regarding alveolar osteitis, is true? A. Generally develops 7-10 days after surgery B. Represents a localized bone infection C. Is theorized to be caused by lysis of a fully formed blood clot prior to its replacement by granulationtissue D. Requires vigorous bone scraping under local anesthesia to stimulate new blood clot formation

Answer: C Rationale: Alveolar osteitis is essentially an inflammation of the bony socket from a recently extracted tooth. Treatment consists of debridement of the socket and placement of a suitable obtundent until the area becomes asymptomatic. Usually no local anesthesia is required. Reference: Ness, GM, Peterson, LJ; Impacted Teeth in Peterson's Principles of Oral and Maxillofacial Surgery, Miloro, M. et al., Editors, Second Edition, BC Decker Inc, 2004, p. 151.

91. In regard to measured periodontal attachment at the distal of the second mandibular molar following extraction and complete healing of the third mandibular molar, the following is true: A. the best results are obtained when guided tissue regeneration techniques are used at the third molar extraction site. B. the best results are obtained when demineralized bone powder is placed within the third molar extraction site. C. third molar extraction sites that undergo no special treatment demonstrate periodontal attachment levels similar to those treated with guided tissue regeneration techniques or demineralized bonepowder. D. use of demineralized bone powder within the third molar extraction site more commonly results in formation of a chronic giant cell reaction and periodontal attachment loss.

Answer: C Rationale: At the time of third molar extraction, different techniques can be implemented in an attempt to improve the periodontal attachment levels distal to the second molar. A study divided patients undergoing third molar extraction into three different perioperative treatment groups (Guided tissue regeneration (GTR), Demineralized bone powder (DMP), and no special treatment). There were 12 patients in each group. There were no statistical differences between any of the treatment groups in probing depth or attachment level around the second molar. Neither GTR nor DMP techniques showed any advantage over simple healing following third molar extraction Reference: Dodson TB, Management of mandibular third molar extraction sites to prevent periodontal defects. JOMS. 2004 Oct; 62(10): 1213-24.

68. A 48-year-old woman presents with a history of metastatic breast carcinoma, previous mastectomy, and radiation to the chest. She continues periodic chemotherapy infusions for control of her disease. Her general dentist extracted tooth #30 six weeks previous to referring the patient to you for a non-healing socket and bone exposure in the area. The most likely cause of the non-healing site is: A. failure to attain primary closure at the time of extraction. B. osteoradionecrosis secondary to the radiation therapy. C. bisphosphonates included in the chemotherapy regimen. D. traumatic extraction with failure of the patient to follow instructed home care.

Answer: C Rationale: Bisphosphonates are widely used in the management of metastatic disease to the bone and in the treatment of osteoporosis. Recently, there is emerging evidence that these drugs are etiologically related to osteonecrosis of the jaws. Answer A is incorrect because primary closure is not possible or required. Although radiotherapy is known to be a cause of osteonecrosis, the stem of the question explicitly indicates radiation to the chest; therefore, option B is incorrect. Option D is also incorrect since even if traumatic extraction of a tooth occurred, healing should have been complete after six weeks, even without appropriate home care. Reference: Ruggiero, S.L., Mehrotra, B., Rosenberg, T.J., and Engroff, S.L.:Osteonecrosis of the Jaws Associated with the use of Bisphosphonates:A Review of 63 cases. J. Oral Maxillofac Surg 62:527-534, 2004.

91. When comparing the apically repositioned flap technique and the closed eruption technique for exposure of an unerupted labially positioned maxillary canine lying high in the alveolus, the closed eruption technique results in: A. less periodontal attachment around the canine. B. a greater risk of intrusive relapse. C. improved gingival esthetics. D. excessive canine clinical crown length.

Answer: C Rationale: Both techniques allow for maintenance of the attached gingiva. In the closed eruption technique, a crestal incision is made, and a full-thickness flap is reflected. Bone is carefully removed around the crown and a bracket attached. The flap is closed and the tooth is allowed to erupt down through the crestal incision site as traction is placed on the ligature. The referenced study, comparing the two techniques, shows that although the periodontal attachment level was the same in the two techniques, the labially impacted maxillary anterior teeth uncovered w/ apical flap techniques had more unaesthetic scarring, increased clinical crown length, and greater risk of intrusive relapse. The results of treatment with the closed technique were aesthetically superior and there was less relapse. Reference: Beck, RJ, Powers, MP; Soft Tissue Considerations in Fonseca, RJ, Oral and Maxillofacial Surgery Vol 7, Reconstruction and Implant Surgery, WB Saunders 2000. p353. Vermette ME: Uncovering labially impacted teeth: Apically positioned flap and closed reduction techniques. Angle Orthodontics 1995; 65: 23-32

91. In order to maintain the crestal bone between adjacent dental implants, the implants should be placed at least how far apart? A. 2.0 mm B. 2.5 mm C. 3.0 mm D. 3.5 mm

Answer: C Rationale: In sites with adjacent implants, bone resorption of 1 to 2 mm at the proximal aspects of the implant leads to a flattening of the inter-implant bone and consequently a short inter-implant papilla. A distance of at least 3 mm has been recommended between 2 adjacent implants to minimize this bone resorption. Reference: Tarnow DP, et al. The effect of inter-implant distance on the height of inter-implant bone crest. J Periodontology 2000; 71:=546-549

22. An adult patient has a self-inflicted shot gun wound to the mouth. He is brought to the emergency room intubated with a large facial wound. Vital signs upon arrival are heart rate 130, respiration 35, and blood pressure 90/60. The patient is fighting restraints. A urinary catheter insertion produced only 10 cc of urine. Which of the following Class of hemorrhage and initial resuscitation treatment is correct? A. Class II. Treat with crystalloid solution B. Class III. Treat with blood transfusion C. Class III. Treat with crystalloid solution and blood transfusion D. Class IV. Treat with crystalloid and blood transfusion.

Answer: C Rationale: Class III hemorrhagic shock is characterized by tachycardia, tachypnea, mental status changes and measurable fall in systolic blood pressure, but without the significant narrowing of pulse pressure seen in Class IV shock. Also the tachycardia and tachypnea are worse in Class IV shock. Lactated Ringers or 0.9% normal saline is the fluid of choice for initial resuscitation. Given excessive hemorrhage, >20%, red blood cells should be replaced to maintain oxygen carrying capacity. Cross-matched is the ideal choice, but time constraints (30 min) may require Type-specific blood (5-15 min) to be used. Reference: Advanced Trauma Life Support for Doctors Course Student Manual, 7th Ed., First Impression Press, 2004, page 75 Miloro M, et al. Peterson's Principles of Oral and Maxillofacial Surgery, 2nd Edition. Pg 343. BC Decker Inc, 204.

68. Which of the following is the best method to determine the completion of facial growth? A. Use of hand/wrist films. B. Completion of changes in skeletal height C. Use of serial lateral cephalograms D. Attainment of maturity by the Tanner classification

Answer: C Rationale: Completion of skeletal height does not correspond to completion of facial growth. A hand- wrist radiograph is inappropriate for assessing facial growth, because it is not specific enough for each patient. The best method of evaluating the completion of facial growth is by superimposing sequential cephalometric radiographs. Most boys do not complete their facial growth until the late teenage years. A 14- or 15-year-old boy may not have gone through his adolescent growth spurt. It is advisable to wait until an adolescent male has completed growth in height. At that point, a cephalometric radiograph should be taken. Another radiograph should be taken at least 6 months to a year later. If these radiographs are superimposed, and there are no changes in vertical facial height (nasion to menton), this indicates that most of the facial growth has been completed. Achievement of Tanner stage V indicates maturity in development of secondary sexual characteristics, but not skeletal maturity. Reference: Kokich V:Orthodontic-restorative management of the adolescent patient. In Orthodontics and Dentofacial Orthopedics, eds McNamara JA Jr. Ann Arbor, MI:=Needham Press; 2001. 425-452. Kokich V:Maxillary lateral incisor implants:=planning with the aid of orthodontics. J Oral Maxillofac Surg. 2004 Sep;62(9 Suppl 2):=48-56. Marshall, W, Tanner, J. Variations in the pattern of pubertal changes in girls. Arch Dis Child 1969; 44:=291 Marshall, W, Tanner, JM. Variations in the pattern of pubertal changes in boys. Arch Dis Child 1970; 45:=13. Law, SJ; Use of Orthopedic appliances in Growth Modification in Fonseca, RJ, Oral and Maxillofacial Surgery Vol 6, Cleft/ Craniofacial/ Cosmetic Surgery, WB Saunders 2000. pp 15-17. Bishara, Facial and dental changes in adolescents and their clinical implications. The Angle orthodontist, 200 Vol 70:=6,p 471

31. Which of the following would be an indication for frontal sinus obliteration? A. Non-displaced greenstick fracture of the anterior table. B. Fracture of the posterior table. C. Nasofrontal duct occlusion. D. Nasolacrimal duct damage.

Answer: C Rationale: Displaced anterior table fractures should be opened with careful inspection of the sinus mucosa. A non-displaced posterior table fracture without CSF leak can be managed without obliteration of the frontal sinus. Occlusion of the nasofrontal ducts requires removal of all sinus mucosa from the frontal sinus, inversion of the mucosa into the ducts, and elimination of dead space. This eliminates the possibility of developing a hematoma or seroma. Fat has been historically used with most success; no filler, hydroxyapatite, glass wool, bone, cartilage, muscle, absorbable gelatin sponge and fabric, acrylic have also been used. Reference: Miloro M, et al. Peterson's Principles of Oral and Maxillofacial Surgery, 2nd Edition. Pg 496-501. BC Decker Inc, 204.

1. Surgical exposure of which of the following areas would require soft tissue re-suspension to re- establish facial form? A. Midface exposure via an oral vestibular incision B. Symphysis exposure via submental incision C. Zygomatic exposure via coronal incision D. Orbital floor exposure via transconjunctival incision.

Answer: C Rationale: Extended exposure of the facial skeleton is indicated for management of many complex facial fractures. Re-suspension of the soft tissue is important to establish proper facial form. Midface exposure via vestibular incision avoids key suspensory areas, thus simple closure is usually indicated. Symphysis exposure via a submental incision leaves the origin of the mentalis muscle attached to the surround soft tissue and/or bone. Re-suspension via this approach is not indicated. With exposure of the orbital floor via a transconjunctival incision simple closure is all that is indicated. When the zygomatic process is exposed via a coronal approach the superficial layer of the temporalis fascia is incised and periosteum overlying the arch is detached. This can result in ptosis of the suborbicularis oculi fat pad. This fat pad must be re-suspended to the lateral orbit. Reference: Manson PN, Clark N, Roberston B, et al Subunet Principles in Midface Fractures:The Importance of Sagittal Buttresses, Soft Tissue Reductions and Sequencing Treatment of Segmental Fractures. Plast Reconstr Surg 103:1287-1306, 1999.

1. A patient presents with a displaced inner and outer table frontal sinus fracture with associated CSF leak. There is no involvement of the nasofrontal drainage system. Which of the following procedures is indicated for the management of the frontal sinus? A. Open reduction and internal fixation of the inner and outer table of the frontal sinus B. Obliteration of the frontal sinus with preservation of the nasofrontal drainage system C. Cranialization of the frontal sinus with obliteration of the nasofrontal draining system D. Obliteration of the frontal sinus with obliteration of the nasofrontal drainage system

Answer: C Rationale: Fracture of the inner and outer table of the frontal sinus with communition and CSF leak represent the most complex of frontal sinus injuries. The dual tears associated with the frontal sinus leak must be repaired and they are usually approached via a coronal incision with the removal of a portion of the frontal bone. With communition of the posterior table ORIF of these fractures would be difficult. Because of the rapid growth of respiratory mucosa any and all remnants of epithelium within the sinus must be removed and the bony walls curetted to remove any epithelium remnants. Obliteration of the sinus is indicated when there is an intact posterior sinus wall. This allows for maintenance of the material used for obliteration to remain within the sinus cavity. In all cases of obliteration of the frontal sinus, obliteration of the nasofrontal ducts is also indicated to prevent respiratory microflora from entering into this cavity or entering into the sinus, thus preventing infection. In this case, due to the fracture of the inner and outer table of the frontal sinus with associated CSF leak, all of the posterior table should be removed to allow for expansion of the frontal lobes into the residual cavity. The outer table is reconstructed in this procedure. In all cases of cranialization of the frontal sinus, the nasofrontal drainage system is obliterated. Reference: Rohrich RJ, Hollier LH. Management of Frontal Sinus Fractures; Changing Concepts. Clinics in Plastic Surgery 19:219-232:1992.

27. Intravenous fluids administered to a patient with acute brain injury should be: A. hypotonic. B. hypertonic. C. normotonic. D. glucose-containing.

Answer: C Rationale: Intravenous fluids administered to acutely brain-injured patients should normotonic in order to maintain normovolemia. Hypovolemia and hypervolemia is harmful to these patients. Hyperglycemia from use of glucose-containing fluids is harmful to the injured brain. Normal saline or Ringer's lactate is recommended for resuscitation. Serum sodium levels need to be monitored to prevent hyponatremia which can lead to brain edema. Reference: Advanced Trauma Life Support for Doctors. Student Course Manual, 7th Edition, p.165. American College of Surgeons 2004.

1. A 16-year-old male is involved in a motor vehicle accident. He is found unresponsive at the scene and is intubated and brought to the emergency room. On arrival he both opens his eyes and withdraws only to painful stimulus. What is his Glascow Coma Scale classification? A. 4T B. 5T C. 6T D. 7T

Answer: C Rationale: Intubated patients receive a 'T' since they are unable to verbally respond Best Eye Response. (4) No eye opening. Eye opening to pain. Eye opening to verbal command. Eyes open spontaneously. Best Verbal Response. (5) 1. No verbal response 2. Incomprehensible sounds. 3. Inappropriate words. 4. Confused 5. Orientated Best Motor Response. (6) 1. No motor response. 2. Extension to pain. 3. Flexion to pain. 4. Withdrawal from pain. 5. Localising pain. 6. Obeys Commands. Reference: ATLS Manual, 1997

57. When performing a maxillary labial frenectomy where the base of the frenum is extremely wide, the most effective surgical technique is: A. Z-plasty technique. B. simple excision ("diamond" excision). C. localized vestibuloplasty with secondary epithelialization. D. kazanjian technique (lip switch).

Answer: C Rationale: Localized vestibuloplasty with secondary epithelialization has been described as the most beneficial for broad frenum attachments. The Z-plasty technique is useful when the mucosal and fibrous band is narrow. The diamond excision can result in scarring and relapse. The lip-switch is used for mandibular vestibuloplasty. Reference: Fonseca, Davis. Reconstructive Preprosthetic Oral & Maxillofacial Surgery (1st Edition). Page 65, W.B Saunders Company, 1986 Philadelphia Ochs, MW, Tucker, MR; Preprosthetic Surgery in Peterson, Ellis, Hupp, Tucker. Contemporary Oral & Maxillofacial Surgery 4th edition. pp 275-278, Mosby, 2003 USA.

68. Which of the following materials used for retrograde fill of an apicoectomy has the lowest success rate? A. IRM (intermediate restorative material) B. SuperEBA (ethoxy benzoic acid) C. Amalgam D. MTA (Mineral Trioxide Aggregate)

Answer: C Rationale: MTA, IRM, and SuperEBA all have success rates greater than that of a retrograde amalgam. Following retrograde preparation with an ultrasonic microtip or use of intraoperative magnification, the success rates of these three materials range from 85-97%. The success rate of traditional retrograde amalgam filling is reported to be much lower, around 68%. Reference: von Arx T. Failed root canals: the case for apicoectomy (periradicular surgery). Journal of Oral & Maxillofacial Surgery. 63(6):832-7, 2005 Jun.

68. When is the most appropriate time to perform a labial frenectomy to facilitate closure of a diastema between teeth numbers 8 and 9? A. Prior to eruption of the maxillary lateral incisors B. Prior to eruption of the maxillary canines C. During eruption of the maxillary canines D. After complete eruption of the maxillary canines

Answer: C Rationale: Maxillary labial frenectomy should be done following the eruption of the lateral incisors and during active eruption of the maxillary canines. The combination of a correctly timed and executed maxillary frenectomy and the anterior eruption force of the maxillary canines should lead to the closure of the diastema. Reference: Kearns, G, Pediatric Dentoalveolar Surgery in Fonseca, RJ, Oral and Maxillofacial Surgery Vol 1, Anesthesia/Dentoalveolar Surgery/ Office Management, WB Saunders 2000. pp 360- 361.

1. When performing a floor-of-the-mouth lowering procedure, it is necessary to: A. perform a subperiosteal dissection B. cover the denuded region with a soft tissue graft C. avoid altering muscle attachments in patients diagnosed with retrolingual sleep apnea D. detach all muscle attachments at the genial tubercle

Answer: C Rationale: Patients with suspected or diagnosed obstructive sleep apnea should not have muscle attachments altered in floor-of -mouth lowering procedures because this may worsen or create obstruction. Supraperiosteal dissections are performed and the incision margin is sutured to the periosteum at the depth of the vestibule. It is not necessary to place a soft tissue graft over the denuded periosteum as this may be allowed to secondarily epithelialize. The genioglossus muscle attachments at the genial tubercle may be partially removed to increase the lingual sulcus, but approximately ½ of the genioglossus attatchment should remain intact to ensure proper tongue function. Reference: Fonseca RJ Oral and Maxillofacial Surgery Vol. 7 p. 49WB Saunders2000

27. Stabilization of the medial canthal tendon during a medial canthopexy should be directed at which of the following directions? A. Anterior and superior B. Anterior and inferior C. Posterior and superior D. Posterior and inferior

Answer: C Rationale: Repair of detached medial canthal tendons during a severe naso-orbital ethmoidal fracture repair involves reattachment of one or both tendons to the posterior lacrimal crest in a slightly posterior and superior direction to overcome the forces of migration, relapse, and telecanthus. Reference: Fonseca, RJ, Oral and Maxillofacial Trauma Vol 2, Diagnosis and Treatment of Midface Fractures, Elsevier 2005

25. When opening a subcondylar fracture through a retromandibular approach: A. the patient should be paralyzed . B. the forehead should always be in view in the surgical field. C. the closure of the parotid capsule/SMAS and platysma layer is critical to minimize the occurrence of a salivary fistula. D. the local anesthesia should be injected deep to the platysma muscle.

Answer: C Rationale: Salivary-cutaneous fistulae are potential sequelae to the retromandibular approach. To help avoid this complication it is recommended that a "water tight" closure be performed. Reference: Ellis E. Zide M. Surgical Approaches to the Facial Skeleton Williams and Wilkins, Media, PA, 1995

91. When considering immediate loading of a dental implant, the minimum insertional torque with which the implant must be placed is: A. 10 - 15 N/cm2. B. 20 - 25 N/cm2. C. 30 - 35 N/cm2. D. 40 - 45 N/cm2.

Answer: C Rationale: Studies that use insertion torque values are in general agreement that the values should be at least 30 to 35 Ncm." Reference: Ganeles, J, Wismeijer, D:Early and Immediately Restored and Loaded Dental Implants for Single-Tooth and Partial-Arch Applications. The International Journal of Oral and Maxillofacial Implants, Volume 19 Supplement, 2004, page 99.

1. At what distance behind the superior aspect of the medial orbital rim would the anterior ethmoidal foramen and its associated artery be located? A. 0 to 5 mm B. 10 to 15 mm C. 20 to 25 mm D. 30 to 35 mm

Answer: C Rationale: The anterior and posterior ethmoidal arteries will require identification when performing medial orbital wall or roof dissections. The vessels may also be identified for ligation in order to control nasal bleeding. The anterior ethmoidal foramen transmits the anterior ethmoidal artery and anterior ethmoidal branches from the nasociliary nerve. The posterior ethmoidal foramen transmits the posterior ethmoidal artery and the spheno-ethmoidal nerve from the nasociliary nerve. These foramina can be located posterior to the junction of the medial orbital wall and orbital roof. The anterior ethmoidal foramen is located twenty to twenty-five millimeters behind the medial orbital rim and the posterior ethmoidal foramen is located twelve millimeters posterior to this point or approximately thirty-two to thirty-seven millimeters. Reference: Ochs MW. Orbital and Ocular Trauma. Miloro M (Ed):Peterson's Principals of Oral and Maxillofacial Surgery, Second Edition. BC Decker, Inc. Hamilton, Ontario 2004. p. 465

36. Rebleeding following a traumatic hyphema: A. is associated with the use of topical cycloplegics. B. can be prevented by increasing intraocular pressure. C. may occur in 5 to 30% of patients. D. is increased with the use of carbonic anhydrase inhibitors.

Answer: C Rationale: The management of a patient with a traumatic hyphema is to use topical cycloplegics and carbonic anhydrase inhibitors to decrease intraocular pressure. Rebleeding can occur in 5- 30% of the patients with hyphema. Reference: Baley JS, Goldwasser MS, Management of Zygomatic complex fractures. In:Miloro M ed, Peterson's Principles of Oral and Maxillofacial Surgery Second edition BC Deker, London 2004 Chapter 23.2 pp 459

91. Which of the following statements is correct regarding the mandibular third molar? A. The tooth germ is usually visible on a radiograph by the age of 6. B. By age 11, the tooth is located within the anterior border of the ramus with the occlusal surface facingsuperiorly. C. The orientation of the crown within the alveolus may be affected by underdevelopment or overdevelopment of its mesial or distal roots. D. The tooth's position from ramus to alveolus is determined strictly by growth of the mandibularbody.

Answer: C Rationale: The mandibular 3rd molar, visible radiographically as a tooth germ usually by age 9 within the anterior border of the ramus, progresses with mandibular growth to the body due to resorption of the anterior border of the ramus. The tooth's anterior facing occlusal surface changes orientation to vertical, usually by age 20, during its root development. The failure to progress to final vertical orientation may be attributed to one or more factors including: < Over or under development of the mesial and/or distal roots < Insufficient arch length to accommodate the normal position of the tooth within the arch Reference: Ness, GM, Peterson, LJ; Impacted Teeth in Peterson's Principles of Oral and Maxillofacial Surgery, Miloro, M. et al., Editors, Second Edition, BC Decker Inc, 2004, pp. 139-140

68. During routine extraction of a maxillary first molar with radiographic periapical pathology, a 5mm fragment of the palatal root is dislodged into the maxillary sinus. What is the most appropriate next step? A. Obtain a CT scan of the maxillary sinus B. Completion of an antibiotic course and observation C. Attempt careful visualization and retrieval through the extraction socket D. Perform a Caldwell-Luc antrostomy

Answer: C Rationale: The most appropriate first step in this situation is to attempt visualization and retrieval of the displaced root tip through the extraction socket. If that is unsuccessful subsequent measures should include further radiographic documentation of the root tip position and discussing the operative findings with the patient. Initial localization with periapical or panorex x-rays would be appropriate. If conservative surgical approaches are unsuccessful, then a Caldwell- Luc may be appropriate. In general, root tips between 1mm and 3mm and are without infection or pathology can be managed expectantly. A root that is 2mm or greater and exhibits infection or pathology should be removed. Reference: Wells, DL, Capes, JO, Powers, MP; Complications of Dentoalveolar Surgery in Fonseca, RJ, Oral and Maxillofacial Surgery Vol 1, Anesthesia/Dentoalveolar Surgery/ Office Management, WB Saunders 2000.p432.

1. A mandibular angle fracture with comminution, infection, or loss of bone buttressing is best treated with which type of fixation: A. miniplate. B. dynamic compression plate (DCP). C. reconstruction plate. D. wire osteosynthesis.

Answer: C Rationale: The need for absolute stability for these types of fractures negates wire or miniplate fixation. DCP causes interfragmentary compression with possible bone devitalization and necrosis. The reconstruction plate is specifically indicated in these instances. Reference: Ellis- Treatment of mandible angle fractures using AO reconstruction plates.JOMS- 1993;51(3):250-254.

1. Which of the following would be observed in a patient with an isolated C4-5 spinal cord injury? A. Disturbance of heart rate B. Apnea C. A major loss of diaphragmatic function D. An air embolism

Answer: C Rationale: The phrenic nerve innervates the diaphragm and arises from cervical segments 3, 4, and 5. Therefore, a cervical spine injury at this level would likely cause severe ventilatory dysfunction, affecting tidal volume, vital capacity, or FEV1. A is incorrect, since parasympathetic innervation to the heart arises from Cranial Nerve X and the sympathetic innervation arises from the cervical sympathetic trunk, and both join to form the cardiac plexus. B is incorrect, since the injury only involves C4-5 (and not C3); the phrenic nerve would probably still be somewhat functional, and some ventilation would be expected. D is incorrect. This is a completely random answer, not necessarily related to this injury. Reference: Hollingshead's Manual of Practical Anatomy

91. What stage of root development is optimal for the surgical uprighting of an impacted mandibular second molar? A. Less than 1/3 root B. 1/3 C. 2/3 D. Complete

Answer: C Rationale: The procedure is best performed after 2/3 of root development is completed. At this stage the risk of root fracture is minimal. Performing this procedure when less than 2/3 of root development has been completed could result in the second molar floating in its new position. Although the procedure has been performed when root development is complete, the incidence of subsequent pulpal necrosis or calcification is increased. Reference: Dessner, S; Surgical uprighting of second molars: rationale and technique: Oral and Maxillofacial Surg Clin N Am 14 (2002) 201-212. Costello, BJ, Malik, AKB, Powers, MP, Fonseca, RJ; Complicated Exodontia in Fonseca, RJ, Oral and Maxillofacial Surgery Vol 1, Anesthesia/Dentoalveolar Surgery/ Office Management, WB Saunders 2000.p240.

1. When performing a Risdon approach to a mandibular angle fracture: A. posterior to the facial artery the marginal mandibular branch of the facial nerve is always located below the inferior border of the mandible. B. posterior to the facial artery the marginal mandibular branch of the facial nerve may drop 2.5cm below the inferior border of the mandible. C. anterior to the facial artery the marginal mandibular branch of the facial nerve is found below the inferior border of the mandible less than 10% of the time. D. anterior to the facial artery the marginal mandibular branch of the facial nerve usually has only one branch.

Answer: C Rationale: The two classic articles on this subject quote incidences of 0% and 6% frequency of the marginal mandibular branch of the facial nerve being found below the inferior border of the mandible anterior to the facial artery. Posterior to the facial artery, 19% of the time the marginal mandibular branch of the facial nerve may be located below the inferior border of the mandible up to 1 (or1.2) cm. Anterior to the facial artery the marginal mandibular branch of the facial nerve has one branch only 21% of the time. Reference: Ziarah HA. Atkinson ME. The surgical anatomy of the mandibular distribution of the facial nerve. [Journal Article] British Journal of Oral Surgery. 19(3):159-70, 1981 Sep Dingman R, Grabb W: Surgical anatomy of the mandibular ramus of the facial nerve based on the dissection of 100 facial halves. Plast Reconstr Surg 29:266-272, 1962

27. Regarding blunt force trauma to the midface, which of the following anatomic structures can be considered a pillar of support that can resist compression and deformation? A. Anterior maxillary sinus wall B. Posterior maxillary sinus wall C. Zygomatico-maxillary junction D. Zygomatico-sphenoid junction

Answer: C Rationale: There are three vertical buttresses of the midface:zygomatico-maxillary, pterygo-maxillary, and naso-maxillary. These structures consist of thicker supporting regions of bone that transmit and/or resist forces to the base of the skull. The horizontal buttresses of the midface are the (1) pyriform aperture, (2) maxillary alveolus and palate, (3) orbital rims and (4) base of the skull. Reference: Fonseca, RJ, Oral and Maxillofacial Trauma Vol 1, Advances in Maxillofacial Trauma Surgery, Elsevier 2005. Manson P, et al. Plastic and Reconstructive Surgery. Structural Pillars of the Facial Skeleton:An Approach to the Management of Le Fort Fractures. July 1980, 66(1) pg. 57.

1. A 79-year-old white male presents to your office for removal of carious teeth. Medical history review reveals chronic obstructive pulmonary disease (COPD), hypertension, peptic ulcer disease, athlerosclerosis with occasional angina, and osteoarthritis. Daily medications include isosorbide dinitrate, furosemide, and acetaminophen. After conscious sedation with midazolam and local anesthesia with prilocaine, you note that in recovery he has slowly become ashen looking and the pulse oximetry reading has fallen to 85%. Which of the following measures is most appropriate? A. Intubation and hyperventilation with 100% oxygen B. Titrated administration of 0.4 mg flumazenil IV C. Methylene blue administration 1 mg/kg IV D. Assisted ventilation by face mask with room air.

Answer: C Rationale: This situation may appear to be pulmonary in origin, but in fact represents acquired methemoglobinemia. This condition can be precipitated by nitrates, (such as isosorbide dinitrate) acetaminophen, prilocaine, articaine, and a number of other medications, especially in genetically susceptible individuals. The oxidized (ferric) state of the methemoglobin molecule cannot be reversed by increasing the FIO2, which also may decrease the respiratory drive in COPD. Sedation reversal by flumazenil will have no effect on the condition. Cautious administration of methylene blue will reduce methemoglobin back to a ferrous state, normalizing the oxygen binding/delivering capacity of hemoglobin. Reference: Benumof JL Anesthesia & Uncommon Diseases, 4th ed. WB Saunders, 1998 pp288-9

1. Which of the following medications may trigger asthmatic symptoms? A. Atropine B. Ipatropium C. Valdecoxib D. Isoetharine

Answer: C Rationale: Valdecoxib (Bextra) is a cyclo-oxygenase-2 inhibitor. Any inhibitor of prostaglandin synthesis (such as nonsteroidal anti-inflammatory drugs) can cause an increase in leukotrienes which cause bronchoconstriction. Atropine, being an anticholinergic, was formerly used to decrease bronchoconstriction in asthma but is no longer used because of its systemic side effects. Ipatropium bromide(Atrovent) is an inhaled anticholinergic used in chronic refractory asthma and in chronic obstructive pulmonary disease. Isoetharine (Bronkosol) is an inhaled B2 agonist used for bronchodilation as a nebulized solution. Reference: Washington Manual of Medical Therapeutics 28th ed., Little Brown, 1995 pp. 238-242

91. Indication for removal of impacted 3rd molars include: A. prevention of mandibular anterior tooth crowding. B. to allow for postoperative regeneration of bone distal to a 2nd molar with chronic periodontitis. C. prior to anticipated sagittal split osteotomy. D. deeply impacted maxillary third molars, prior to anticipated LeFort I osteotomy.

Answer: C Rationale: While somewhat controversial, crowding of mandibular incisor teeth appears to be associated with deficient arch length rather than the mere presence of impacted teeth. After 3rd molar removal the bone height distal to the 2nd molar usually remains at the pre-operative level. Removal of 3rd molars at mandibular advancement osteotomy reduces the thickness and quality of lingual bone at the proximal aspect of the distal segment where fixation screws are usually applied. During a LeFort osteotomy, removal of deeply impacted maxillary 3rd molars from the maxillary sinus side may be done safely without compromising the soft tissue vascular pedicle of the maxilla. Reference: Ness, GM, Peterson, LJ; Impacted Teeth in Peterson's Principles of Oral and Maxillofacial Surgery,Miloro, M. et al., Editors, Second Edition, BC Decker Inc, 2004, pp. 140-153.

1. When performing a z-plasty to remove a prominent labial frenum the secondary incisions are made at an angle approximately 60 degrees to allow the main limb to be rotated: A. 33 degrees B. 45 degrees C. 60 degrees D. 90 degrees

Answer: D Rationale: A z-plasty is designed to rotate the frenum or scar 90 degrees. Secondary incisions made at other angles may not allow as great a rotation of the main limb (in this case, the main frenum incision) as those made at 60 degrees tothe main limb. Reference: Fonseca, RJ, Oral and Maxillofacial Surgery Vol 7, Reconstruction and Implant Surgery, WB Saunders 2000

31. Which of the following is the most common early complication related to frontal sinus trauma? A. Mucocele formation B. Brain abscess C. Mucopyocele formation D. Meningitis

Answer: D Rationale: All of the above cited answers are potential complications to frontal sinus fractures. Meningitis is the most common EARLY complication. The remainders of the answers are delayed or late potential complications. Frontal sinus complications can occur many years after injury (often greater than 20 years after injury). The most serious complications: can include displacement of the frontal bone in the brain and floor of frontal sinus into the orbit. Infectious complications, such as meningitis or intracranial abscess occur via bacterial spread through posterior table fractures or the diploe veins of Behçet; or by osteomyelitis, or mucocoele formation. Mucoceles are the most common chronic complication with diagnosis made via CT scanning. Periodic review by CT should be done every 1, 2 and 5 years following surgery. Symptoms include headache and can include loss of smell (anosmia.). Reference: Fonseca, RJ, Oral and Maxillofacial Trauma, Third Edition, 2005, Vol 2, pp 731-32. Miloro M, et al. Peterson's Principles of Oral and Maxillofacial Surgery, 2nd Edition. Pg 503. BC Decker Inc, 204.

36. The most predictable means to treat a large posterior auricular skin/cartilage avulsion is: A. reattachment as a composite graft with hyperbaric oxygen. B. microvascular reimplantation. C. full thickness skin graft to cover exposed cartilage. D. 2 phase bipedicled postauricular flap.

Answer: D Rationale: Exposed cartilage will not accept a skin graft. Composite free auricular grafts and microvascular reanastomosis have poor success rates due to poor venous drainage. The bipedicled flap nicely and predictably reconstructs the posterior helical defect. If the avulsed segment is 1 cm or less it can be reattached and allowed to revascularize. Larger avulsive injuries, the "pocket principle" can be used where the detached ear is dermabraded and reattached to the stump and then buried under a skin flap in the posterior auricular region to provide vascular supply. 2-3 weeks later the revascularize ear is uncovered and allowed to reepithelialize. Reference: Dierks EJ, Deeb GR, Evaluation and management of ear injuries, OMS Knowledge Update vol 3, AAOMS, 2001

1. Which of the following is the treatment of choice in management of bilateral mandibular body fractures in a patient with an extremely atrophic mandible (less than 6 mm of bone height)? A. Closed reduction with a gunning splint B. Lag screw fixation of the mandibular segments C. Fixation with mini plates D. Fixation with reconstruction plates

Answer: D Rationale: In order to allow healing of a fracture of the atrophic mandible, the fixation technique must neutralize the tension forces on the mandible. The only fixation technique that satisfies these criteria is the use of a reconstruction plate. Reference: Sikes JW, Smith BR, Mukherjee DP. An In-Vitro Study of the Effect of Bony Buttressing on Fixation Strength of a Fractured Atrophic Edentulous Mandible Model J Oral Maxillofacial Surgery 58:56-61, 2000.

91. The osteotome technique for implant placement: A. is primarily used in the mandible. B. is used in Type I bone. C. compromises vascularity. D. compresses bone laterally.

Answer: D Rationale: In soft maxillary bone, preparation of the implant osteotomy site can be completed using a series of osteotomes rather than burs. Because the maxillary bone is soft, an osteotome can be used to split or widen a narrow ridge to receive an implant. In addition, the osteotome condenses and laterally compresses the soft bone at the osteotomy site, placing a denser, compressed bone immediately adjacent to the implant.. Reference: Fonseca, RJ, Oral and Maxillofacial Surgery Vol 7, Reconstructive and Implant Surgery, WB Saunders, 2000, pp 111-114

57. One of the most critical complications involved in genial tubercle reduction is: A. over-reduction of the tubercle. B. under-reduction of the tubercle. C. partial detachment of the genioglossus muscle. D. hematoma of the floor of the mouth.

Answer: D Rationale: Reduction of the genial tubercle is performed to create an appropriate base for a denture when significant mandibular resorption has occurred. Under reduction will not provide the desired alveolar surface, over reduction may result in complete muscle detachment and difficulty swallowing for several months until reattachment occurs. The genioglossus muscle is partially detached or may require complete detachment to adequately reduce the tubercle, it will re attach independently. Floor of the mouth hematoma may cause airway embarrassment and an emergent situation. Reference: Spagnoli, DB, Gollehon, SG, Misiek, DJ; Preprosthetic and Reconstructive Surgery in Peterson's Principles of Oral and Maxillofacial Surgery, Miloro, M. et al., Editors, Second Edition, BC Decker Inc, 2004, pp169-170. Davis et al. Soft Tissue Procedures in Reconstructive and Preprosthetic Surgery, Fonseca and Davis Editors, Second Edition. WB Saunders Co., 1995. pp748, 759-760

68. Which of the following statements is true regarding the Bio-Col ridge preservation technique? A. Primary closure is necessary B. Requires use of some autogenous bone graft C. Should not be used in the esthetic zone D. Can be used with immediate or delayed implant placement

Answer: D Rationale: Site preservation usually begins at the time of tooth removal. In aesthetic areas, the referenced author uses the Bio-Col alveolar ridge preservation technique to preserve hard and soft tissue alveolar ridge anatomy in preparation for immediate or delayed implant placement. This technique can be used to reduce or avoid osseous ridge resorption by minimizing trauma during tooth removal. The prepared extraction sockets or the voids surrounding the immediately placed implant are then grafted with Bio-Oss (Osteohealth, Shirley, NY), a natural, porous bone-grafting material. Subsequently, the grafted socket is isolated with an absorbable collagen material (Collaplug; Zimmer Dental, Carlsbad, CA) that has been coated with an impervious tissue cement (Isodent; Ellmann International, Hewlett, NY); this allows for guided bone regeneration without the need for flap elevation and primary closure, thus preserving the surrounding soft-tissue volume. Finally, the scalloped soft tissue architecture is preserved with the use of interim provisional restorations, anatomic healing abutments, or custom tooth-form healing abutments designed to support the marginal tissues and interdental papillae. This is of critical aesthetic importance for implant patients who present with thin scalloped periodontal biotypes who are predisposed to loss of alveolar ridge volume secondary to remodeling, resorption of bone following tooth removal, and soft tissue recession following subsequent surgical or restorative interventions. Reference: Sclar AG, Strategies for management of single-tooth extraction sites in aesthetic implant therapy. J Oral N:Maxillofac Surg. 2004 Sep;62(9 Suppl 2) 90-105. Sclar AG, Preserving alveolar ridge anatomy following tooth removal in conjunction with immediate implant placement. The Bio-Col technique. Atlas Oral Maxillofac Surg Clin North Am. 1999 Sep;7(2): 39-59.

91. Soft tissue peri-implantitis occurs most frequently with which of the following implant restorations? A. Single tooth replacement B. Implant and tooth supported fixed prosthesis C. Implanted supported fixed prosthesis D. Implant supported overdenture

Answer: D Rationale: Soft tissue peri-implantitis occurs most commonly in association with implant supported overdentures, with a reported frequency of 11% to 32%. Rates of soft tissue peri-implantitis associated with implant supported fixed prostheses range from 7% to 20%. Reference: Ardekian, L; Dodson, TB, Complications associated with the placement of dental implants:Oral and Maxillofacial Surg Clin NAm15 (2003) 243-249

91. A 20-year-old presents with a radiolucency associated with an impacted tooth. What is the most common pathologically significant lesion detected on histopathologic examination of the pericoronal tissue? A. Odontogenic keratocyst B. Ameloblastoma C. Odontoma D. Dentigerous cyst

Answer: D Rationale: The cited study reported the histopathologic diagnoses of a large series of pericoronal lesions in adults submitted to an oral and maxillofacial pathology biopsy service. Of the 2646 lesions submitted, 33% showed demonstrable pathology. The most common pathology was dentigerous cyst (28.4%), followed by OKC (3%), odontoma (0.7%), and ameloblastoma (0.5%). Reference: Curran A, Pathologically significant pericoronal lesions in adults: Histopathologic evaluation. JOMS. 2002 Jun; 60(6): 613-7

68. A 70-year-old diabetic man who has a history of left sinus symptoms was referred to you for extraction of erupted and loose tooth #15. Your examination revealed marked necrosis of osseous and soft tissue of the left maxilla. You removed several teeth, the necrotic bone and soft tissue, and submitted the specimen to pathology. The pathologist reported necrotic bone and soft tissue with inflammatory infiltrate and the presence of nonseptate irregularly wide fungal hyphae with frequent right- angle branching. Your diagnosis is: A. Aspergillosis. B. Candidiasis. C. Actinomycosis. D. Mucormycosis.

Answer: D Rationale: The correct answer to this question requires specific knowledge of the morphology of fungi. Aspergillus forms fruiting bodies and septate filaments branching at acute angles. Candida grows as yeast forms, tandem arrays of elongated forms without hyphae (pseudohyphae), and true hyphae with septate. Actinomyces is a gram-positive filamentous or rod-shaped bacteria. Mucormycosis form nonseptate, irregularly wide fungal hyphae with frequent right-angle branching. In addition mucormycosis frequently occur in diabetics and may spread from nasal sinuses to the orbit and brain-giving rise to rhinocerebral mucormycosis. Reference: Regezi:Oral Pathology:Clinical Pahtologic Correlations, 4th., 2003 Elsevier, pp 37-38. Strauss, R.A.:Fungal Infections of the Head and Neck, In:Oral & Maxillofacial Surgery Clinics of North America. Vol. 3, Number 2, W. B. Saunders 1991, pp 295-309.

1. When the medial canthal ligament is attached to a bony segment in naso-orbito-ethmoidal(NOE) fracture repair the transcanthal wire is best placed: A. after all soft tissue injuries have been addressed. B. anterior to the original insertion of the canthal ligament. C. posterior and inferior to the original insertion. D. posterior and superior to the original insertion.

Answer: D Rationale: The purpose of the trans-canthal wire is to secure the canthal ligament and boney segment in the pretraumatic position. Pull of the soft tissues displaces the bone and canthal ligament in an anterior and inferior direction. Therefore a wire placed posterior and superior to the original insertion provides a vector whose resistance to displacement is most ideal and provides the best alignment. Reference: OMS Knowledge Update, Volume three, Section 6. Abubaker AO and Strauss RA, eds. p TRA 75-76. Classification D - Trauma - Soft tissue Oral and Maxillofacial Surgery In- Training Examination (OMSITE) questions for the Trauma Section.

36. In order to avoid irreversible vision loss, an orbital fracture associated with optic nerve compression must be addressed within which length of time? A. 24 hours B. 36 hours C. 4 hours D. 2 hours

Answer: D Rationale: The retina is an extension of the brain, and will have similar reaction in the presence of ischemia. Signs and symptoms of visual impairment, severe ocular pain should be evaluated for retrobulbar hematoma. Computed tomography evaluation can demonstrate signs of retrobulbar hematoma formation, but immediate treatment with a lateral canthotomy based on clinical symptoms and signs should be completed prior to a lengthy CT scan. Other signs of optic nerve compromise include (1) afferent papillary defect (2) visual field loss and (3) color vision defects.

91. On the above scan, what is the location of the third molar inadvertently displaced during an attemptedextraction? A. Superficial temporal space B. Maxillary sinus C. Pterygomaxillary fissure D. Infratemporal space

Answer: D Rationale: The tooth clearly is not in the maxillary sinus. Additionally is not in the pterygomaxillary fissure between the pterygoid plate and posterior maxilla. The level of the scan is below the superficial temporal space, the inferior boundary of which is the zygomatic arch. The tooth resides in the infratemporal space. The infratemporal space is the portion of the deep temporal space that lies inferior to the infratemporal crest of the sphenoid bone. The inferior border is the superior surface of the lateral pterygoid muscle. The anterior border of the deep temporal space is composed of the posterior wall of the maxillary sinus, the pterygomaxillary fissure, and the posterior surface of the orbit, including the inferior orbital fissure. Reference: Flynn, TR In Topazian, RG, Goldberg, MH, Hupp, JR, editors: Oral and Maxillofacial Infections, ed 4, Philadelphia, 2002, W B Saunders Co., pp 200-203.

91. 7 days after third molar extraction, a patient presents with swelling and trismus. The large contrast-enhancing fluid collection on the above scan is best described as residing in which of the following spaces? A. Submental B. Masseteric C. Sublingual D. Pterygomandibular

Answer: D Rationale: This is an axial scan of a pterygomandibular space abscess. The scan reveals a radiolucent, contrast-enhancing collection between the ascending ramus of the mandible and distended medial pterygoid muscle. The lateral pharyngeal space and airway are distorted by mass effect and edema. The borders of the pterygomandibular space are the ascending ramus of the mandible laterally, the medial pterygoid muscle medially, the pterygomasseteric sling inferiorly, and the lateral pterygoid muscle superiorly. The parotid gland and capsule form the posterior border. The pterygomandibular raphe is the anterior border. REFERENCE: Flynn , TR In Topazian , RG, Goldberg, MH, Hupp, JR, editors: Oral and Maxillofacial Infections, ed 4, Philadelphia, 2002, W B Saunders Co., pp 188-213.

57. Treatment for inflammatory fibrous hyperplasia where areas of gross tissue redundancy are present, is best treated by excision with: A. electrosurgical technique. B. laser technique. C. primary closure. D. secondary epithelialization.

Answer: D Rationale: When areas of gross tissue redundancy are found, excision frequently results in total elimination of vestibule. In such cases excision of the epulides, with peripheral mucosal repositioning and secondary epithelialization, is preferred. Reference: Ochs, MW, Tucker, MR; Preprosthetic Surgery in Peterson, Ellis, Hupp, Tucker. Contemporary Oral & Maxillofacial Surgery 4th edition. Page 274, Mosby, 2003 USA.

57. Cervical spine fractures concomitant to maxillofacial injury are most often: A. "burst' fractures. B. "hangman" fractures. C. Jefferson fractures. D. subluxations.

Answer: D Rationale: When considering maxillofacial injury and cervical spine injury, a cause and effect relationship exists between the mandible and cervical spine. There is also a relationship between neurocranial injury and mid- and upper-facial third facial injuries. Approximately one out of every fifty patients with a mandibular fracture will suffer a cervical spine fracture. The vast majority of these are subluxation injuries. When the mandible sustains a force applied to it, the reaction of the body is to move away from the force. This "twisting of the neck, when accentuated, results in a subluxation injury of the cervical spine. Reference: Haug RH, Wible RT, Likavec MJ, et al:Cervical spine fractures and maxillofacial trauma. J Oral Maxillofac Surg 49:725-729, 1991.

53. The cause of permanent visual loss subsequent to traumatic hyphema is: A. elevated intraocular pressure. B. retinal artery spasm. C. traumatic mydriasis. D. corneal staining.

Answer: D Rationale: While concomitant injury has been associated with hyphema and permanent visual loss, it is not the primary cause in traumatic hyphema. Retinal artery spasm is not associated with hyphema. Both increased intraocular pressure and traumatic mydriasis are concomitant problems. Corneal staining from myoglobin is the primary cause of permanent loss of vision. Reference: Brandt MT, Haug RH:Traumatic Hyphema: A Comprehensive Review. J Oral Maxillofac Surg. 59:1462-1470, 2001.


Ensembles d'études connexes

Gateway A2. Unit 5. Grammar revision

View Set

PMP Rita Exam Prep Questions - Chapter 10 - Communications Mgmt

View Set

Conditions Existing Before Conception

View Set